LSAT and Law School Admissions Forum

Get expert LSAT preparation and law school admissions advice from PowerScore Test Preparation.

User avatar
 Dave Killoran
PowerScore Staff
  • PowerScore Staff
  • Posts: 5850
  • Joined: Mar 25, 2011
|
#80551
This game is also discussed in our Podcast: LSAT Podcast Episode 70: The May 2020 LSAT-Flex Logic Games Section

Complete Question Explanation
(The complete setup for this game can be found here: viewtopic.php?t=33052)

The correct answer choice is (A).

Answer choice (A): This is the correct answer choice.

Answer choice (B): This answer is incorrect because it violates the fourth rule of the game by not having an HM block.

Answer choice (C): This answer is incorrect because it violates the third rule of the game by an OM block.

Answer choice (D): This answer is incorrect because it violates the fifth rule of the game by having not having a performance of J before the first performance of H.

Answer choice (E): This answer is incorrect because it violates the second rule of the game by having the 2nd and 7th performances be different.

Get the most out of your LSAT Prep Plus subscription.

Analyze and track your performance with our Testing and Analytics Package.